Thread Rating:
  • 0 Vote(s) - 0 Average
  • 1
  • 2
  • 3
  • 4
  • 5
nbme question - smite1
#1
A 2-month-old infant is brought to the emergency department by paramedics after his mother found him not breathing in his crib. He is apneic and cyanotic; no pulses are palpable, but he is warm. After manually stabilizing his neck and positioning the airway, which of the following is the most appropriate next step in management?
A) Positive pressure ventilation and oxygenation
B) Two chest thrusts followed by chest compressions
C) Establishing vascular access
D) Administration of epinephrine
E) Electrical defibrillation

Answer key says A. But why cant it be E. Kaplan Emergency Medicine guy says that do AED since vfib is most common cause of arrest rythm. Airway and CPR after AED.

Reply
#2
This is pulselessness. Causes are asystole, vF, vT, PEA.
There are no indication that it a arrythmia.

Crib = hypoxia » apneic + cyanosis ,
Mostl likely dx : PEA Tx underlying cause.

Approach should be
1. Airwar (done)
2. Breathing (Vent)
3.circulation (iv acsess, if indicated, after iv acsess: epi)

So answer is A. Am I right?
Reply
#3
ok thx. make sense. forgot about baby being in the "crib"

can u help me with this one....

A previously healthy 42-year-old woman comes to the physician because of a 6-month history of shortness of breath with moderate exertion and pulsatile tinnitus in her left ear. She has had to decrease her daily running from 40 minutes to 20 minutes at a slower pace. She takes no medications. Her last menstrual period was 5 weeks ago. She is sexually active and uses a diaphragm for contraception. She appears well. Her blood pressure is 130/60 mm Hg. pulse is 96/min, and respirations are 16/min. A bruit is heard over the left ear. The lungs are clear to auscultation. A grade 216, early systolic murmur is heard at the left sternal border. The cardiac apex is laterally displaced and enlarged. Peripheral pulses are bounding. An ECG shows no abnormalities except for left ventricular hypertrophy. Echocardiography shows a left ventricular ejection fraction of 50% and an increased left ventricular end-diastolic volume.
Which of the following is the most likely diagnosis?

O A) Aortic stenosis
O B) Arteriovenous malformation
O C) Hypertrophic cardiomyopathy
O D) Osteitis deformans (Paget's disease)
O E) Pregnancy
O F) Vitamin B1 (thiamine) deficiency

Answer key says B. Y not C
Reply
#4
This is hyperdynamic circulation causing early sysrolic murmur. So discarding AS ( late systolic ejection murmur) , Unlikely that 6 months pregnancy, paget can cause hyperdynamic circulation, but with no pulsatile mass. Hypertrophied LV can lead to CHF.
A-V fistula matches then most.
Does it sound fair?
Reply
#5
ok. i guess HOCM wouldnt really cause a bounding pulse. i appreciate the help!!
Reply
#6
43. A previously healthy 37-year-old man comes to the physician because of intermittent pain and swelling of his left knee over the past 3 months. He takes no medications. He has been sexually active only with his girlfriend for the past 2 years and uses condoms consistently. He drinks one to two beers each week and does not smoke or use illicit drugs. He is a landscape architect and lives and works on Long Island, New York. His temperature is 37°C (98.6°F), blood pressure is 120180 mm Hg, and pulse is 801min. Examination of the left knee shows edema, erythema, and warmth. The remainder of the examination shows no abnormalities. Arthrocentesis yields slightly cloudy fluid with a leukocyte count of 6000/mm3 (80% segmented neutrophils and 20% lymphocytes). Examination of the fluid shows no crystals, and Gram's stain shows no organisms. Which of the following is the most likely causal organism?

O A) Borrelia burgdorfen
O B) Neisseria gonorrhoeae
O C) Parvovirus
O D) Staphylococcus aureus
O E) Streptococcus pneumoniae
O F) Treponema pallidum

Answer key says A.

Im a little confused with this question. Generally in septic arthritis leukocyte count is >50,000. For counts between 5000-50,000, its usually inflammatory conditions (ie. RA) or gout/pseudogout. Is lyme arthritis an exception? Can someone explain this concept???? Couldnt find it in the books.
Reply
#7
MTB 2 says lyme arthritis has about 25000 WBC/mm3 in joint fluid..n its difficult to distinguish it from other inflammation or infection.. serology testing is required to prove it.. only thing that favors lyme is that he is from newyork..
Since he is having intermittent painn sweeling .. can it be parvovirus???
Reply
#8
do u guys have answers for NBME 4?? will really appreciate
Reply
#9
thanks catchther

ya i do. whats your email address? im actually looking for the nbme form 4 itself. perhaps we could trade!!
Reply
#10
So with ABC now changed to CAB, is this still A? I've been trying hard to find if there are any exceptions for SIDS but it's been really tough.
Reply
« Next Oldest | Next Newest »


Forum Jump: